LSAT and Law School Admissions Forum

Get expert LSAT preparation and law school admissions advice from PowerScore Test Preparation.

 jessicamorehead
  • Posts: 84
  • Joined: Jul 07, 2017
|
#44313
So that's why I had such a hard time deciding which was correct.... because they are essentially the same thing. I'm laughing in the middle of Starbucks hahaha. Thanks, Jamena!!
User avatar
 jwooon
  • Posts: 32
  • Joined: Jun 15, 2024
|
#107520
Hi,

I understand this question and why the answer is (B), but what I am confused about is the diagramming of (A). Without diagramming, I was still able to eliminate it pretty fast, but I just want to straighten up my diagramming.

The way I diagrammed it is:
HC :arrow: A
A :arrow: WV
:longline:
HC :arrow: WV

where:
HC = Huang's collection
A = paintings put up for auction
WV = wide variety of artists

In my diagram, I feel like the argument is valid and straightforward, but when I don't diagram, I noticed that the argument isn't really valid? Firstly, "Huang's collection" implies (although not in LSAT terms) that all the art in the collection is by one artist (therefore, not a wide variety of artists). Secondly if Huang's Collection is only a part of all the art being submitted, then couldn't all the other arts be by a variety of artists but not Huang's collection?

When I draw a visual diagram of the relationship, it seems as though HC is within the parameters of A, but if all A :arrow: WV, then wouldn't that condition apply to HC? Or is WV not a condition? Also what exactly is a premise test?
 Adam Tyson
PowerScore Staff
  • PowerScore Staff
  • Posts: 5376
  • Joined: Apr 14, 2011
|
#107599
As I see it, jwooon, the first problem with answer A is that the second premise is not a conditional statement. When the author says "the paintings to be auctioned next week are by a wide variety of artists," there is no element of "if this, then that." It's just an absolute fact - there is a wide variety. Conditional statements aren't absolute like that; the necessary condition is only guaranteed when a sufficient condition occurs. But there doesn't seem to be a sufficient condition in that statement, and there doesn't seem to be anything necessary, either.

To show you why that isn't conditional, let's take your diagram of A :arrow: WV and put it back into words:

If a painting is in the auction next week, it is by a wide variety of artists.

Is that what they meant? Probably not. Wouldn't it make more sense if each painting was by a single artist, but that there are a lot of different artists with paintings in the auction?

Aside from that, answer A doesn't match the stimulus because the conclusion is not a contrapositive.

Finally, in your diagram, the "A" ultimately ends up meaning two different things. In the first premise, it means "included in the auction," while in the second premise, it means the auction as a whole, not just something that is in the group.

Also, it may be worth noting that "Huang Collection" does not imply a single painter. Huang might be the owner of the collection, and there could be multiple artists in that collection.
User avatar
 jwooon
  • Posts: 32
  • Joined: Jun 15, 2024
|
#107606
I see. Just one quick question: in parallel questions, does contrapositive not equal to the original conditional? I assumed that since negative doesn't matter, any conditional and its contrapositive were the same conditional.

For example, if the stimulus has a reasoning of A :arrow: B and (not) B :arrow: (not) A, I thought I could pick an AC that has the reasoning A :arrow: B and A :arrow: B.
User avatar
 Dana D
PowerScore Staff
  • PowerScore Staff
  • Posts: 385
  • Joined: Feb 06, 2024
|
#107721
Hey Jwooon,

What answer choice are you referring to here? The question asks us for the argument that is 'most similar' to the stimulus, so I would say if you found an answer choice that is the contrapositive of the argument, answer choice (B) would still be the best answer because it exactly mirrors the stimulus' argument.
User avatar
 jwooon
  • Posts: 32
  • Joined: Jun 15, 2024
|
#107722
Hi Dana,

Thank you for all your help with my questions. I was asking more of a general question about parallel reasoning/parallel flaw questions. When there is a stimulus that contains an argument that diagrams like A :arrow: B, (not)B :arrow: A (so a contrapositive pair), I tend not to eliminate answers that have diagrams like (A :arrow: B, A :arrow: B) because contrapositives are functionally identical to the original statement.

Is it an acceptable basis to eliminate and answer choice just because the stimulus contains a contrapositive and the AC doesn't?

Hope this clarifies!
User avatar
 Dana D
PowerScore Staff
  • PowerScore Staff
  • Posts: 385
  • Joined: Feb 06, 2024
|
#107775
Hey Jwoon,

Speaking generally, I would say no, don't eliminate an answer choice just because it is a contrapositive. However, usually in parallel flaw questions you will see the exact same type of logic pattern. If you do narrow it down to 2 answer choices and one perfectly matches and the other uses a contrapositive, then pick the one that more closely matches, since parallel flaw questions are asking for an answer choice that most closely matches the stimulus. Does that make sense? That being said, I don't think this happens often (if at all) on the test.
User avatar
 jwooon
  • Posts: 32
  • Joined: Jun 15, 2024
|
#107806
Yes, sounds good. Thank you!

Get the most out of your LSAT Prep Plus subscription.

Analyze and track your performance with our Testing and Analytics Package.